ММО-55, II даваа, Дунд ангийн багш

Дунд ангийн багш   

Бодлогын тоо: 6    Хугацаа: 540 мин


1. $n\ge2$, $k\ge2$ гэе. $a_1,\dots,a_n\ge0$ тоонуудын арифметик дунджийг $A$ гэвэл $$2^{k-1}\left(|a_1-a|^k+\dots+|a_n-A|^k\right)\le\left(|a_1-A|+\dots+|a_n-A|\right)^k$$ тэнцэтгэл биш биелэхийг харуул.

Заавар Бодолт
Заавар.

Бодолт. $y1,\dotsc, y_\ell\ge0$ ба $ z_1,\dotsc, z_m\ge0$ хувьд $y_1+\dots+y_\ell = z_1 +\dots + z_m = B$ гэе. Энэ үед $$(y^k_11 + \dots + y^k_{\ell}) + (z^k_1 + \dots + z^k_m) \le (y_1 + \dots + y_{\ell})^k + (z_1 + \dots + z_m)^k = 2B^k$$ болно. Эндээс бодлого хялбар мөрдөнө. Зөвхөн $(A + a, A - a, A, A, . . . , A)$ хэлбэртэй дарааллын сэлгэмэл дээр тэнцэтгэл биелнэ.


2. $n\ge 2$ гэе. $1,2,\dots,n$ дугаартай $n$ зогсоол дээр $1,2,\dots,n$ дугаартай $n$ машин байрлуулав. Машин бүрийн дугаараас уг машин зогссон зогсоолын дугаарыг хасахад гарах $n$ ялгавар бүгд ялгаатай байвал энэ байрлуулалтыг зөв гэж нэрлэе. Нийт зөв байрлуулалтын тоо сондгой гэж батал.

Заавар Бодолт
Заавар.

Бодолт. $i$ дугаартай зогсоолыг $n + 1 − i$ гэж шинээр дугаарлахад зогсоолууд мөн л $1,2,\dots,n$ тоогоор дугаарлагдана. Зөв байрлуулалтын хувьд машин бүрийн дугаар дээр уг машин зогссон зогсоолын шинэ дугаарыг нэмэхэд гарах $n$ нийлбэр бүгд ялгаатай байна. Учир нь k дугаартай i зогсоолд зогссон машины шинэ зогсоолын дугаар ба машины дугаарын нийлбэр $n + 1 + (k − i)$ юм. Иймд машины болон зогсоолын дугаарын нийлбэрүүд бүгд ялгаатай байх байрлалын тоог сондгой гэж үзүүлэхтэй манай бодлого ижил юм. Энэ чанартай байрлалыг цаашид зөв гэе. $i$ дугаартай зогсоолд $x_i$ дугаартай машин зогссон байрлалыг $(x_1, \dots, x_n)$ гэж тэмдэглэе. $(1, \dots , n)$ байрлал зөв учраас ямар нэг $i$ дугаарын хувьд $x_i\neq i$ байх зөв байрлалын тоог тэгш гэж үзүүлэхэд хангалттай. $(x_1,\dots, x_n)$ байрлал өгөгдсөн үед $x_i$ дугаартай зогсоолд $i$ дугаартай машиныг зогсоох замаар шинэ байрлал байгуулъя. Хэрэв анхны байрлал зөв бол шинэ байрлал мөн зөв байна. Одоо энэ хоёр байрлал ялгаатай гэдгийг үзүүлье. $(x_1,\dots, x_n)$ зөв учраас $x_i\neq i$ байх ямар нэг $i$ дугаар олдоно. Энэ байрлалын $i$ дугаартай машин $y_i$ дугаартай зогсоолд зогссон гэвэл $x_i \neq y_i$ байна. Эсрэг тохиолдолд $i + x_i = y_i + i$ болж зөрчил гарна. Шинээр байгуулсан зөв байрлалын $i$ дугаартай зогсоолд $y_i$ дугаартай машин зогсох тул $(x_1,\dots, x_n)$-с ялгаатай. Үүгээр бодлого бодогдов.


3. $ABC$ гурвалжныг багтаасан тойрог $\omega$ ба $\measuredangle B>\measuredangle C$ гэе. $\omega$ тойргийн $A$ цэгийг агуулах $BC$ нумын дундаж цэгийг $T$ ба $ABC$ гурвалжинд багтсан тойргийн төвийг $I$ гэе. $\omega$ тойрог дотор орших $E$ цэгийн хувьд $AE\parallel BC$ ба $\measuredangle AEI=90^\circ$ байг. $TE$ шулууны $\omega$ тойргийг огтлох шинэ цэг нь $P$ ба $\measuredangle B=\measuredangle IPB$ бол $\measuredangle A$ өнцгийг ол.

Заавар Бодолт
Заавар.

Бодолт. $\omega$ тойргийн төв нь $O$ ба $TM$ нь $\omega$ тойргийн диаметр байг. $PI$ шулууны $\omega$ тойргийг огтлох шинэ цэгийг $Q$ гэе. Тэгвэл $M$ нь $BC$ нумын дундаж цэг байх ба $A$, $I$, $M$ нэг шулуун дээр оршино. $AE\parallel BC$ ба $\angle AEI = 90^\circ$ гэдгээс $IE\perp BC$ байна. Мөн $TM\perp BC$ тул $IE\parallel TM$ болно. Иймд $\angle AIE = \angle AMT = \angle APT$ гэдгээс $A$, $P$, $I$, $E$ нэг тойрог болно. Эндээс $\angle AP Q =\angle AP I = 180^\circ - \angle AEI = 90^\circ$ буюу $A$, $O$, $Q$ нэг шулуун дээр оршино. Иймд $\angle BP I = \angle BP Q = \angle BAQ = 90^\circ - \angle ACB$ тул $\angle B = \angle BP I = 90^\circ - \angle ACB$ буюу $\angle B + \angle C = 90^\circ$ гэдгээс $\angle A = 90^\circ$ байна.


4. $2019$-н тоог аль ч дараалсан таван $a$, $b$, $c$, $d$, $e$ тооных нь хувьд $a - b + c - d + e = 55$ байхаар тойрог дээр байрлуулж болдог бол уг тоонуудыг ол.

Заавар Бодолт
Заавар.

Бодолт. Тоонуудаа $a_0, a_1,\dots, a_{2018}$ гээд дурын $i\ge 0$ хувьд $x_i = a_i - 55\pmod{2019}$ гэж тэмдэглэе. Өгсөн нөхцөлөөс дурын $i \ge 0$ хувьд $x_i - x_{i+1} + x_{i+2} - x_{i+3} + x_{i+4} = 0$ болно. Мөн ижлээр $x_{i+1} - x_{i+2} + x_{i+3} - x_{i+4} + x_{i+5} = 0$ тул $x_{i+5} = -x_i$ байна. Эндээс дурын $k \ge 0$ хувьд $x_{i+5k} = (-1)^kx_i$ болох ба $k = 404$ гэвэл $x_{i+2020} = x_i$ болно. Иймд $x_{i+1} = x_{i+2020} = x_i$ болох тул $x_i$ тогтмол болох ба өгсөн нөхцөлөөс $x_i = 0$ болно. Энэ нь $a_0 = a_1 = \dots = a_{2018} = 55$ гэсэн үг


5. $n \ge 2$ гэе. $n$ зэргийн бодит $P(x)$ олон гишүүнт ба $m \ge 1$ урттай тогтмол биш $a_1 ,\dots, a_m$ арифметик прогресс $P(a_1 ),\ldots, P (a_m)$ дараалал мөн тогтмол биш арифметик прогресс болохоор олддог байх $m$ тооны хамгийн их утгыг ол.

Заавар Бодолт
Заавар. $n$-ээс хэтрэхгүй зэргийн олон гишүүнт нь $n+1$ цэг дээрх утгаараа нэг утгатай (интерполяцийн олон гишүүнт) тодорхойлогддог.

Бодолт. $(a_1,aa_1+b), (a_2,aa_2+b),\ldots,(a_m,aa_m+b)$ цэгүүдийг дайрсан олон гишүүнтийн зэрэг $n$ нь $m$-ээс бага бол заавар ёсоор олдох цор ганц олон гишүүнт нь $P(x)=ax+b$ байна. Энэ нь $n=\deg P(x)\ge 2$ гэсэнд зөрчинө. Иймд $m\le n$ байна. Нөгөө талаас $$P(x)=(x-1)(x-2)\dots(x-n)+x$$ гэсэн $n$ зэргийн олон гишүүнт бодлогын нөхцөлийг хангана. Энэ тохиолдолд $m=n$ тул $m$-ийн боломжит хамгийн их утга нь $n$ юм.


6. $n^m = m^n + m + n$ байх бүх натурал тоон $(n, m)$ хосыг ол.

Заавар Бодолт
Заавар.

Бодолт. $(n, m) = (2, 5)$ шийд болох нь илт тул өөр шийдгүй гэж харуулъя.

$n = 1$ үед $1^m < m^1 + m + 1$ тул шийдгүй. $n = 2$ үед $m \le 4$ шийдгүй болохыг шалгахад төвөггүй. Индукцээр $m \ge 6$ хувьд $$2^m > m^2 + m + 2\qquad (*)$$ гэж харуулъя. $m = 6$ үед $2^6 = 64 > 44 = 6^2 + 2 + 6$ үнэн ба $m \ge 6$ хувьд үнэн гэвэл $$2^{m+1} \ge 2 \cdot (m^2 + m + 2)= m^2 + m^2 + 2m + 4> m^2 + 3m + 4= (m + 1)^2 + (m + 1) + 2$$ тул $m + 1$ хувьд үнэн. Иймд $n = 2$, $m \neq 5$ шийдгүй.

$n \ge 3$ гээд дахиад тохиолдолд салгая. $m = 1$ үед $n^1 < 1^n + 1 + n$ тул шийдгүй.

$m = 2$ үед дурын $n \ge 1$ хувьд $n^2 < 2^n + 2 + n$ байна. Үнэхээр, $n \le 5$ хувьд шалгахад төвөггүй ба $n \ge 6$ хувьд $(*)$–с $$2^n + 2 + n > 2^n > n^2 + n + 2 > n^2$$ байна. Иймд $m = 2$ үед шийдгүй.

$m \ge 3$ гэе. $m > n \ge 3$ хувьд $$n^m > m^n + m + n\qquad (**)$$ гэж харуулъя. Эхлээд $e = \sup\limits_{n\ge 1}\left(1 +\dfrac{1}{n}\right)^n< 2.72 <\dfrac{74}{27}$ болохыг саная. $m = n + 1$ үед $$\dfrac{1}{n^n}\big((n + 1)^n + (n + 1) + n\big) = \left(1 +\dfrac{1}{n}\right)^n+\dfrac{2}{n^{n-1}}+\dfrac{1}{n^n}\le e +\dfrac{2}{3^2}+ \dfrac{1}{3^3}< 3\le n$$ тул үнэн. Одоо $m > n$ хувьд үнэн гээд $m + 1$ тохиолдлыг авч үзье. Энд $$n \ge 3 \ge e \ge\left(1 +\dfrac{1}{m}\right)^m\ge\left(1+\dfrac{1}{m}\right)^n$$ болохыг анзаарвал $$n^{m+1} > n(m^n + m + n) > (m + 1)^n + n(m + n) > (m + 1)^n + (m + 1) + n$$ болно. Индукцээр дурын $m > n \ge 3$ хувьд үнэн.

Эцэст нь $n \ge m \ge 3$ гэе. $n = m$ үед илт шийдгүй ба $n > m \ge 3$ үед $(**)$–с $$m^n + m + n > m^n > n^m + n + m > n^m$$ болох тул шийдгүй.